february · • the regulating act of 1773 created the office with the title of governor-general...

61
PRE-Mix February 2020 Visit our website www.sleepyclasses.com or our YouTube channel for entire GS Course FREE of cost Also Available: Prelims Crash Course || Prelims Test Series

Upload: others

Post on 04-Apr-2020

3 views

Category:

Documents


0 download

TRANSCRIPT

Page 1: February · • The Regulating Act of 1773 created the office with the title of Governor-General of the Presidency of Fort William, or Governor-General of Bengal to be appointed

P R E-M i x February

2020

Visit our website www.sleepyclasses.com or

our YouTube channel for entire GS Course FREE of cost

Also Available: Prelims Crash Course || Prelims Test Series

Page 2: February · • The Regulating Act of 1773 created the office with the title of Governor-General of the Presidency of Fort William, or Governor-General of Bengal to be appointed

1. Geography 1 ....................................................................................................

2. History 9 ............................................................................................................

3. Polity 20 ...............................................................................................................

4. International Relations 28 ............................................................................

5. Economy 32 ........................................................................................................

6. Environment 43 ................................................................................................

7. Science & Tech 53.............................................................................................

Table of Contents

Page 3: February · • The Regulating Act of 1773 created the office with the title of Governor-General of the Presidency of Fort William, or Governor-General of Bengal to be appointed

1. Geography To watch the following questions on YouTube, click on the links given below

• Video 1

• Video 2

• Video 3

• Video 4

1. Choose the correct reasons for differential heating of land and water:

1. Specific heat of water is much greater than the land.

2. Oceanic areas are generally clouded and therefore they receive less insolation than land surface.

3. Reflection of insolation from the oceanic water surface is far more than from the land surface.

4. Opacity of land is high when compared to water.

Select the Correct Code

A. 1 and 2 only

B. 2,3 and 4 only

C. 1 ,3 and 4 only

D. 1,2,3 and 4

Answer: D

2. Consider the following statements about the Roaring Forties

1. They blow an interrupted in the Northern and Southern hemisphere.

2. They blow with great strength and constancy.

3. The direction is generally from North West to east in the southern hemisphere.

4. Overcast skies rain and raw weather are generally associated with them.

 Which of these statements are correct?

A. 1, 2 and 3 only

B. 2,3 and 4

C. 1,3 and 4 only

D. 1,2 and 4

Answer: B

www.YouTube.com/SleepyClasses www.SleepyClasses.com

1

Page 4: February · • The Regulating Act of 1773 created the office with the title of Governor-General of the Presidency of Fort William, or Governor-General of Bengal to be appointed

3. Recently Triple Billion targets has been in news sometimes related to

A. WHO

B. UNEP

C. UNESCO

D. UNGC

Answer: A

Explanation

WHO ‘Triple Billion’ targets:

It aims for:

1. One billion more people to be benefitted from Universal Health Coverage (UHC)

2. One billion more people to be protected from health emergencies

3. One billion more people to be covered for better health and well-being

4. Port Blair is located on which of the following Islands?

A. South Andaman

B. Middle Andaman

C. North Andaman

D. Great Nicobar

Answer: A

5. TulBul Project is associated with which of the following rivers?

A. Indus

B. Ravi

C. Jhelum

D. Beas

Answer: C

6. Which of the following is true in context of “Avartansheel Kheti”?

1. It insists that farmers should produce food for themselves first and what remains after consumption should be sold in the market.

2. It involves dividing the land into zones, for cereal crops, vegetables, fruiting trees and animal husbandry.

Select the Correct Option

A. Only 1

www.YouTube.com/SleepyClasses www.SleepyClasses.com

2

Page 5: February · • The Regulating Act of 1773 created the office with the title of Governor-General of the Presidency of Fort William, or Governor-General of Bengal to be appointed

B. Only 2

C. Both 1 and 2

D. Neither 1 nor 2

Answer: C

Explanation:

• Avartansheel Kheti  is based on the philosophy of A Nagraj, who was a proponent of harmonious co-existence.

• It can be translated as ‘periodic proportionate farming.’

• It insists that farmers should produce food for themselves first and what remains after consumption

should be sold in the market.

• This way, the farmer will never put harmful pesticides and fertilizers if they are growing for themselves.

• It involves dividing the land into zones, for cereal crops, vegetables, fruiting trees and animal husbandry.

Actual technique

• This technique requires a farmer to divide his land into three parts.

Part I

• This part of the farm is required to grow  trees, timber, and fruits. The cost of labor required to maintain this part of the land is low.

• The diversity of the crops helps to maintain ecological balance.

• This part of the land also gives the farmer dried leaves which can be later used for creating compost to increase the soil fertility.

Part II

• This area in the farm can be used to rear livestock.

• Milk by the cattle can be used by the family and the excess can be used to make products like cottage

cheese for higher profits.

• Also, animal dung can be used as manure which reduces the farm’s dependence on external chemicals.

Part III

• This land is made use of to grow crops for the household.

• Staples like wheat, rice along with pulses, cereals, vegetables, fruits, and spices are grown.

7. Which among the following Island of the Andaman & Nicobar islands contains the only known examples of mud volcanoes in India, called locally as ‘Jalki'?

A. Baratang Island

www.YouTube.com/SleepyClasses www.SleepyClasses.com

3

Page 6: February · • The Regulating Act of 1773 created the office with the title of Governor-General of the Presidency of Fort William, or Governor-General of Bengal to be appointed

B. Barren Island

C. Car Nicobar

D. Havelock Island

Answer: A

Explanation

Baratang contains the only known examples of mud volcanoes in India. These mud volcanoes have erupted sporadically, with recent eruptions in 2005 believed to have been associated with the 2004

Indian Ocean earthquake. The previous major eruption recorded was on 18 February 2003. The locals call this mud volcano jalki.

8. Musandam Peninsula is associated with?

A. Turkey

B. Oman

C. Egypt

D. Djibouti

Answer: B

Explanation

www.YouTube.com/SleepyClasses www.SleepyClasses.com

4

Page 7: February · • The Regulating Act of 1773 created the office with the title of Governor-General of the Presidency of Fort William, or Governor-General of Bengal to be appointed

9. Which of the following rivers serves as a nesting ground for Olive Ridley turtles?

A. Devi River

B. Pennar River

C. Hughli River

D. Bhima River

Answer: A

Explanation:

Debi river/Devi River is the one of the main distributaries of Mahanadi River. It flows through Odisha and joins Bay of Bengal. It also serves as nesting ground for Olive Ridley turtles.

10.The UNESCO Creative Cities Network (UCCN) was created in 2004 to promote cooperation with and

among cities that have identified creativity as a strategic factor for sustainable urban development. Which of the following cities in India associated with Gastronomy?

A. Jaipur

B. Hyderabad

C. Chennai

D. Lucknow

Answer: B

Explanation:

INDIAN CONTEXT:

1. Jaipur-Crafts and Folk Arts (2015).

2. Varanasi-Creative city of Music (2015).

3. Chennai-Creative city of Music (2017).

4. Mumbai – Film (2019).

5. Hyderabad – Gastronomy (2019).

11.Mankidia Tribe recently seen in news are the Particularly Vulnerable Tribal Groups (PVTG) present in

which of the following States

A. Odisha

B. Rajasthan

C. Arunachal Pradesh

D. Kerala

Answer: A

www.YouTube.com/SleepyClasses www.SleepyClasses.com

5

Page 8: February · • The Regulating Act of 1773 created the office with the title of Governor-General of the Presidency of Fort William, or Governor-General of Bengal to be appointed

12.Conuco, Ray and Masole are associated with which of the following types of cultivation?

A. Shifting Cultivation

B. Commercial Farming

C. Plantation Agriculture

D. Nomadic Herding

Answer: A

13.Which among the following correctly describes the term watershed?

A. An area drained by a river and its tributaries.

B. The pattern of flow of water in a river channel over a year.

C. The boundary line separating one drainage basin from the other.

D. None of the above.

Answer: C

14.Which of the following best describes the process of Lithification?

A. It is process of formation of rocks through compaction of sediments.

B. It is the process of erosion of rocks and their deposition in deltaic plains.

C. It refers to the cooling of magma to form lava plains.

D. It refers to the metamorphosis of rocks under heat and pressure.

Answer: A

15.Most of the Jute Mills of India are located in which state:

A. Chhattisgarh

B. West Bengal

C. Assam

D. Orissa

Answer: B

16.Kappaphycus alvarezii has been in news recently. Which of the following is true in context with the

same?

1. It is an invasive seaweed which smothers and kills coral reefs.

2. It has recently been found in coral reef areas of Valai island in the Gulf of Manner.

Select the Correct Option

A. Only 1

B. Only 2

www.YouTube.com/SleepyClasses www.SleepyClasses.com

6

Page 9: February · • The Regulating Act of 1773 created the office with the title of Governor-General of the Presidency of Fort William, or Governor-General of Bengal to be appointed

C. Both 1 and 2

D. Neither 1 nor 2

Answer: C

17.Consider the following pairs:

Peninsula Country

1. Anatolian Peninsula: Italy

2. Sinai Peninsula: Egypt

3. Kamchatka Peninsula: Russia

Which of the pairs given above is/are correctly matched?

A. 1 only

B. 1 and 3 only

C. 2 and 3 only

D. 1, 2 and 3

Answer: C

Explanation:

• Anatolian Peninsula: Turkey

• Sinai Peninsula : Egypt

• Kamchatka Peninsula: Russia

18.The tribes “Bedouins“ are associated with which of the following climatic region?

A. Polar

B. Equatorial rainforest

C. Desert

D. Monsoon

Answer: C

Explanation

Bedouin, also spelled Beduin, Arabic Badawi and plural Badw, Arabic-speaking nomadic peoples of the Middle Eastern deserts, especially of North Africa, the Arabian Peninsula, Egypt, Israel, Iraq, Syria, and

Jordan

19. Identify the crop:

1. It is a tropical plantation crop.

2. 16°–28°C temperature, 150-250cm rainfall and well-drained slopes are essential for its growth.

www.YouTube.com/SleepyClasses www.SleepyClasses.com

7

Page 10: February · • The Regulating Act of 1773 created the office with the title of Governor-General of the Presidency of Fort William, or Governor-General of Bengal to be appointed

3. It grows on hilly slopes at the height of 900-1800m.

4. Low temperature, frost, dry weather for a long time and harsh sunshine are harmful for its plants.

Select the Correct Option

A. Coffee

B. Tobacco

C. Jute

D. None of the above

Answer: A

Explanation:

Debi river/Devi river is the one of the main distributaries of Mahanadi River. It flows through Odisha

and joins Bay of Bengal. It also serves as nesting ground for Olive Ridley turtles.

20.Puncak Jaya is the only place where ice glaciers can be found in Southeast Asia. It is located in :

A. Vietnam

B. Indonesia

C. Brunei

D. Laos

Answer: B

www.YouTube.com/SleepyClasses www.SleepyClasses.com

8

Page 11: February · • The Regulating Act of 1773 created the office with the title of Governor-General of the Presidency of Fort William, or Governor-General of Bengal to be appointed

2. History To watch the following questions on YouTube, click on the links given below

• Video 1

• Video 2

• Video 3

• Video 4

1. Which of the following is/are true about Governor-General?

1. The Regulating Act of 1773 created the office with the title of Governor-General of the Presidency

of Fort William, or Governor-General of Bengal

2. The Government of India Act 1813 re-designated the office with the title of Governor-General of

India.

A. 1 only

B. 2 only

C. Both 1 and 2

D. Neither 1 nor 2

Answer: A

Explanation:

• The Regulating Act of 1773 created the office with the title of Governor-General of the Presidency of

Fort William, or Governor-General of Bengal to be appointed by the Court of Directors of the East India Company (EIC).

• The Court of Directors assigned a Council of Four (based in India) to assist the Governor General, and decision of council was binding on the Governor General during 1773-1784.

• The Government of India Act 1833 re-designated the office with the title of Governor-General of

India.

2. After the resignation of Subhas Bose in April 1939, who took over as the President of the INC?

A. J L Nehru

B. Maulana Abul Kalam Azad

C. Pattabhi Sitaramaiya

D. Dr. Rajendra Prasad

Answer: D

Explanation:

• For the 1939 elections of the President of Congress, Subhash announced his candidature knowing

that he would be opposed.

www.YouTube.com/SleepyClasses www.SleepyClasses.com

9

Page 12: February · • The Regulating Act of 1773 created the office with the title of Governor-General of the Presidency of Fort William, or Governor-General of Bengal to be appointed

• When Nehru returned from Europe in 1938, Gandhi suggested him to announce his name as a candidate. But he declined and suggested the name of Maulana Azad.

• But Maulana Azad withdrew his name and then new name came up was of Dr. Pattabhi Sitaramayya, an Andhra leader. 

• Bose contested and won.

• Eventually, due to differences with Gandhiji which led to his differences with the CWC, Bose resigned from the president’s post in April 1939.

• This led to the election of Rajendra Prasad as president of the Congress.

3. In 1942, __________ called Bose the “Prince among the Patriots”.

A. JL Nehru

B. Sardar Patel

C. C Rajagopalachari

D. None of the above

Answer: D

Explanation:

In 1942, Gandhi called Bose the “Prince among the Patriots”. When the death of Bose was reported, Gandhi said that Netaji’s “patriotism is second to none.”

4. Which of the following is/are true?

1. As per the CR Plan, one of the points was that the League shall cooperate with Congress in forming

a provisional government at centre.

2. Jinnah was opposed to having a common centre.

A. 1 only

B. 2 only

C. Both 1 and 2

D. Neither 1 nor 2

Answer: C

Explanation:

The main points in the CR Plan were:

• Muslim League to endorse Congress demand for independence.

• League to cooperate with Congress in forming a provisional government at centre.

• After the end of the war, the entire population of Muslim majority areas in the North-West and North-East India to decide by a plebiscite, whether or not to form a separate sovereign state.

www.YouTube.com/SleepyClasses www.SleepyClasses.com

10

Page 13: February · • The Regulating Act of 1773 created the office with the title of Governor-General of the Presidency of Fort William, or Governor-General of Bengal to be appointed

• In case of acceptance of partition, agreement to be made jointly for safeguarding defence, commerce, communications, etc.

• The above terms to be operative only if England transferred full powers to India.

Objections

• Jinnah wanted the Congress to accept the two-nation theory.

• He wanted only the Muslims of North-West and North-East to vote in the plebiscite and not the entire population.

• He also opposed the idea of a common centre.

5. Statement A: The British government sent a high-powered mission of three British cabinet members

to India to find out ways and means for a negotiated, peaceful transfer of power to India. Pethick

Lawrence was the chairman of the mission

Statement B: The traditional bulwarks of the Raj, such as the bureaucracy and the army, were also now sympathizing with the Indians.

A. Statements A and B are both true, and Statement B is the reason for A

B. Statements A and B are both true, and Statement B is not the reason for A

C. Statement A is correct, and Statement B is incorrect

D. Statement B is correct, and Statement A is incorrect

Answer: A

Explanation:

• The Attlee government announced in February 1946 the decision to send a high-powered mission of three British cabinet members (Pethick Lawrence, Secretary of State for India; Stafford Cripps,

President of the Board of Trade; and A.V. Alexander, First Lord of Admiralty) to India to find out ways and means for a negotiated, peaceful transfer of power to India.

• Pethick Lawrence was the chairman of the mission

Reasons for Imminent Withdrawal

1. Nationalism had penetrated into hitherto untouched sections and areas

2. There was a demonstration in favour of nationalism among the bureaucracy and the loyalist sections; because the paucity of European ICS recruits and a policy of Indianisation had ended the British

domination of the ICS as early as the First World War and by 1939, there existed a British-Indian

parity. After the war, a depleted, war-weary bureaucracy battered by the 1942 events remained.

3. Demands of leniency for INA prisoners from within the Army and the revolt of the RIN ratings had

raised fears that the armed forces may not be as reliable.

6. Which of the following is/are true about Subhash Bose?

1. He joined the Indian Civil Service but resigned to join the Indian freedom movement.

www.YouTube.com/SleepyClasses www.SleepyClasses.com

11

Page 14: February · • The Regulating Act of 1773 created the office with the title of Governor-General of the Presidency of Fort William, or Governor-General of Bengal to be appointed

2. He had also been the mayor of the city of Calcutta.

Select the Correct Option

A. 1 only

B. 2 only

C. Both 1 and 2

D. Neither 1 nor 2

Answer: C

Explanation:

• SC Bose passed the Indian Civil Services examination securing fourth position but resigned from the

service in 1921 to join the struggle for freedom by becoming a member of the Congress.

• His political guru was Chittaranjan Das.

• He became mayor of Calcutta in 1923.

7. Statement A: The Japanese handed over the Indian prisoners of war to Mohan Singh who tried to recruit them into an Indian National Army.

Statement B: With the Japanese contemplating an Indian invasion, the idea of an armed wing of INA seemed more relevant to them.

A. If both Statements A and B are correct, and B is the reason for A

B. If both Statements A and B are correct, and B is not the reason for A

C. If Statement A is correct, but Statement B is incorrect

D. If Statement B is correct, but Statement A is incorrect

Answer: A

Explanation:

• The idea of creating an army out of the Indian prisoners of war (POWs) was originally that of Mohan

Singh, an Indian army officer who had decided not to join the retreating British army in Malaya. He

decided to turn to the Japanese for help.

• The Japanese handed over the Indian prisoners of war to Mohan Singh who tried to recruit them into

an Indian National Army.

• After the fall of Singapore, several POWs were ready to join Mohan Singh. By the end of 1942, 40,000

men were ready to join the INA.

• In September 1942, the first division of the INA was formed with 16,300 men.

• With the Japanese contemplating an Indian invasion, the idea of an armed wing of INA seemed more

relevant to them.

www.YouTube.com/SleepyClasses www.SleepyClasses.com

12

Page 15: February · • The Regulating Act of 1773 created the office with the title of Governor-General of the Presidency of Fort William, or Governor-General of Bengal to be appointed

8. Which of the following statements is/are correct about SC Bose?

1. On October 21, 1943, Subhash Bose formed the Provisional Government for Free India at

Singapore

2. This provisional government declared war on Britain and the United States and was recognised by

the Axis powers.

Select the Correct Option

A. 1 only

B. 2 only

C. Both 1 and 2

D. Neither 1 nor 2

Answer: C

Explanation:

• On October 21, 1943, Subhash Bose formed the Provisional Government for Free India at Singapore with H.C. Chatterjee (Finance portfolio), M.A. Aiyar (Broadcasting), Lakshmi Swaminathan (Women

Department), etc.

• The famous slogan—“Give me blood, I will give you freedom” was given in Malaya.

• This provisional government declared war on Britain and the United States, and was recognised by

the Axis powers.

• Recruits were trained and funds collected for the INA.

9. Which of the following is/are true?

1. The Wavell Plan of July 1945 failed to break the constitutional deadlock between the Congress and the League

2. In June 1945, Labour Party formed the government in Britain. Clement Attlee took over as the new prime minister.

Select the Correct Option

A. 1 only

B. 2 only

C. Both 1 and 2

D. Neither 1 nor 2

Answer: D

Explanation:

• The Wavell Plan (June 1945) backed by the Conservative government in Britain failed to break the

constitutional deadlock.

www.YouTube.com/SleepyClasses www.SleepyClasses.com

13

Page 16: February · • The Regulating Act of 1773 created the office with the title of Governor-General of the Presidency of Fort William, or Governor-General of Bengal to be appointed

• In July 1945, Labour Party formed the government in Britain. Clement Attlee took over as the new prime minister and Pethick Lawrence as the new secretary of state for India.

• In August 1945, elections to central and provincial assemblies were announced.

10.Statement A: The nationalist sentiment which reached a crescendo around the INA trials developed

into violent confrontations with the authority in the winter of 1945-46 in the form of three major

upsurges

Statement B: The Congress, including Gandhiji, did not officially support these upsurges because of their tactics and timing.

A. Statements A and B are both true, and Statement B is the reason for A

B. Statements A and B are both true, and Statement B is not the reason for A

C. Statement A is correct, and Statement B is incorrect

D. Statement B is correct, and Statement A is incorrect

Answer: A

Explanation:

The nationalist sentiment which reached a crescendo around the INA trials developed into violent confrontations with the authority in the winter of 1945-46.

There were three major upsurges—

1. November 21, 1945—in Calcutta over the INA trials

2. February 11, 1946—in Calcutta against the sevenyear sentence to INA officer Rashid Ali

3. February 18, 1946—in Bombay, strike by the Royal Indian Navy ratings

• The Congress did not officially support these upsurges because of their tactics and timing.

• Negotiations had been an integral part of the Congress strategy, to be explored before a mass movement could be launched, especially when the British were seen to be preparing to leave soon.

• In Gandhi’s opinion, the mutiny was badly advised: if they mutinied for India’s freedom, they were

doubly wrong; if they had any grievances, they should have waited for the guidance of leaders.

11.Who said that the “Congress is tottering to its fall, and one of my great ambitions, while in India, is to

assist it to a peaceful demise”?

A. Lord Curzon

B. Lord Dufferin

C. Lord Minto

D. None of these

Answer: A

www.YouTube.com/SleepyClasses www.SleepyClasses.com

14

Page 17: February · • The Regulating Act of 1773 created the office with the title of Governor-General of the Presidency of Fort William, or Governor-General of Bengal to be appointed

Explanation:

• In 1900; Lord Curzon announced to the Secretary of State, that "the Congress is tottering to its fall,

and one of my great ambitions, while in India, is to assist it to a peaceful demise".

• Lord Curzon the 'arch proconsul of imperialism' remarked "My own belief is that the Congress is

tottering to its fall, and one of my great ambitions while in India is to assist it to a peaceful demise“.

12.Who started the English weekly ‘New India’?

A. Lala Lajpat Rai

B. Bipin Chandra Pal

C. Dada Bhai Naoroji

D. Madan Mohan Malviya

Answer: B

Explanation:

• New India (Weekly) was started by Bipin Chandra Pal

• New India (Daily) was started by Mrs. Annie Besant

13.Which of the following is not true about the Muslim League?

A. It was established by the Nawab Salimullah.

B. It was established in Calcutta in 1906.

C. The league supported the partition on Bengal

D. The league opposed the Swadeshi movement

Answer: B

Explanation: The Muslim League was started in Dhaka in 1906.

14.Which of the following provisions were included in the Nehru Report?

1. India must be given Dominion status

2. The Governor-General must be only the constitutional head

3. There was to be no separate electorate

Select the Correct Option

A. 1 and 2 only

B. 2 and 3 only

C. 1 and 3 only

D. All of the above

Answer: D

www.YouTube.com/SleepyClasses www.SleepyClasses.com

15

Page 18: February · • The Regulating Act of 1773 created the office with the title of Governor-General of the Presidency of Fort William, or Governor-General of Bengal to be appointed

Explanation:

• Dominion status on lines of self-governing dominions as the form of government desired by Indians

• Rejection of separate electorates which had been the basis of constitutional reforms so far; instead, a demand for joint electorates with reservation of seats for Muslims at the Centre and in provinces

where they were in minority (and not in those where Muslims were in majority, such as Punjab and

Bengal) in proportion to the Muslim population there with right to contest additional seats.

• The Indian Parliament at the Centre to consist of a 500-member House of Representatives elected on

the basis of adult suffrage, a 200-member Senate to be elected by provincial councils; the House of Representatives to have a tenure of 5 years and the Senate, one of 7 years; the central government to

be headed by a governor-general, appointed by the British government but paid out of Indian

revenues, who would act on the advice of the central executive council responsible to the Parliament.

15.Consider the following statement(s) related to the Indian National Congress.

1. The Poorna Swarajya resolution was passed in Lahore session of the Congress held in December

2. The Congress Working Committee,-which met on January 2, 1930, decided that January 26,1930,

should be observed as the Poorna Swarajya Day.

Select the Correct Option

A. 1 only

B. 2 only

C. Both 1 and 2

D. Neither 1 nor 2

Answer: C

Explanation:

• Declaration of the Independence of India, was promulgated by the Indian National Congress on 19

December 1929, resolving the Congress and Indian nationalists to fight for Purna Swaraj, or complete

self-rule independent of the British Empire.

• The flag of India was hoisted by Jawaharlal Nehru on 31 December 1929 on the banks of Ravi river, in

Lahore, modern-day Pakistan.

• The Congress asked the people of India to observe 26th of January as Independence Day

16.Which of the following is/are true about the food of Harappans?

1. There is archaeological evidence for cultivation of pea, chickpea, pigeon pea, horse gram and green gram.

2. Oilseeds such as sesame, linseed, and mustard were also grown.

3. Animal foods were consumed only in very small quantities.

Select the Correct Option

www.YouTube.com/SleepyClasses www.SleepyClasses.com

16

Page 19: February · • The Regulating Act of 1773 created the office with the title of Governor-General of the Presidency of Fort William, or Governor-General of Bengal to be appointed

A. 1 and 2 only

B. 2 and 3 only

C. 3 and 4 only

D. All of the above

Answer: A

Explanation:

• To judge from the quantity of bones left behind, animal foods were consumed in abundance: beef,

buffalo, mutton, turtles, tortoises, gharials, and river and sea fish,” food historian K T Achaya recorded in his magisterial history of Indian food, Indian Food: A Historical Companion

• Indus Valley Civilisation grew and ate a variety of cereals and pulses. There is archaeological evidence

for cultivation of pea (matar), chickpea (chana), pigeon pea (tur/arhar), horse gram (chana dal) and green gram (moong).

• There is evidence that the Harappans cultivated Italian millet, ragi and amaranth, as well as sorghum and rice. Achaya writes that oilseeds such as sesame, linseed, and mustard were also grown.

17.Which of the following is/are true about RIN Mutiny?

1. The ratings at shore establishments in Karachi, Madras, Calcutta, Mandapam, Visakhapatnam, and the Andaman Islands also joined the revolting ratings of Bombay.

2. While the immediate trigger was the demand for better food and working conditions, the agitation soon turned into a wider demand for independence from British rule.

3. One of the triggers for the RIN strike was the arrest of a rating, BC Dutt

Select the Correct Option

A. 1 and 2 only

B. 2 and 3 only

C. 3 and 4 only

D. All of the above

Answer: D

Explanation:

• On February 18, 1946, some 1,100 Indian sailors or “ratings” of the HMIS Talwar and the Royal Indian Navy (RIN) Signal School in Bombay declared a hunger strike, triggered by the conditions and

treatment of Indians in the Navy.

• The morning after February 18, somewhere between 10,000-20,000 sailors joined the strike, as did shore establishments in Karachi, Madras, Calcutta, Mandapam, Visakhapatnam, and the Andaman

Islands.

• While the immediate trigger was the demand for better food and working conditions, the agitation

soon turned into a wider demand for independence from British rule. www.YouTube.com/SleepyClasses

www.SleepyClasses.com 17

Page 20: February · • The Regulating Act of 1773 created the office with the title of Governor-General of the Presidency of Fort William, or Governor-General of Bengal to be appointed

• One of the triggers for the RIN strike was the arrest of a rating, BC Dutt, who had scrawled “Quit India” on the HMIS Talwar.

18.Which of the following is are true about the Rohillas?

1. The Rohillas were Afghans who entered India in the 18th century as the Mughal Empire was in

decline, and took control of Rohilkhand

2. In 1748, Nawab Ali Muhammad Khan, the adopted son of the Rohilla chief assisted Ahmad Shah Durrani in his conquest of India

Select the Correct Option

A. 1 only

B. 2 only

C. Both 1 and 2

D. Neither 1 nor 2

Answer: C

Explanation:

• The Rohillas were Afghans who entered India in the 18th century as the Mughal Empire was in

decline, and took control of Rohilkhand, at the time known as Katehr.

• In 1737, Nawab AIi Muhammad Khan received the territory of Katehr(Rohilkhand) from Emperor

Muhammad Shah, only to lose everything to Nawab Wazir of Oudh in 1746.

• Two years later, he assisted Ahmad Shah Durrani in his conquest of India, recovering all his former

possessions.

• The Rampur royals have played an important role in the socio-cultural history of the Ganga-Yamuna belt. They run the Amir Raza library in Rampur, once known as the official darbar of the Nawab,

which is home to some 15,000 manuscripts in Arabic, Urdu, Persian and Turkish, as well as a seventh-century Quran.

19.Statement A: He completed translation of fifty Upanishads from their original Sanskrit into Persian in

1657 so that they could be studied by Muslim scholars.

Statement B: Dara Shikoh developed a friendship with the seventh Sikh Guru,  Guru Har Rai and devoted much effort towards finding a common mystical language between Islam and Hinduism

A. Both statements A and B are correct, and B is the reason for A

B. Both statements A and B are correct, and B is not the reason for A

C. Statement A is correct, and Statement B is incorrect

D. Statement B is correct, and Statement A is incorrect

Answer: A

www.YouTube.com/SleepyClasses www.SleepyClasses.com

18

Page 21: February · • The Regulating Act of 1773 created the office with the title of Governor-General of the Presidency of Fort William, or Governor-General of Bengal to be appointed

Explanation:

• Dara Shikoh was a follower of Lahore's famous Qadiri Sufi saint Mian Mir, whom he was introduced to

by Mullah Shah Badakhshi (Mian Mir's spiritual disciple and successor).

• Mian Mir was so widely respected among all communities that he was invited to lay the foundation

stone of the Golden Temple in Amritsar by the Sikhs.

• Dara Shikoh subsequently developed a friendship with the seventh Sikh Guru, Guru Har Rai.

• Dara Shikoh devoted much effort towards finding a common mystical language between Islam and

Hinduism.

• Towards this goal he completed the translation of fifty Upanishads from their original Sanskrit into

Persian in 1657 so that they could be studied by Muslim scholars.

• His translation is often called Sirr-e-Akbar ("The Greatest Mystery"), where he states boldly, in the introduction, his speculative hypothesis that the work referred to in the Qur'an as the "Kitab al-

maknun" or the hidden book, is none other than the Upanishads.

20.Which of the following statements is/are correct?

1. “The 21st” in Bangla, commemorates the day in 1952 when students of the University of Dhaka

launched a nationwide protest against the imposition of Urdu on the people of what was then East Pakistan.

2. In recognition of the Bengali people’s struggle for their language and culture, UNESCO announced that January 21 would be observed worldwide as International Mother Language Day.

Select the Correct Option

A. 1 only

B. 2 only

C. Both 1 and 2

D. Neither 1 nor 2

Answer: B

Explanation:

• Ekushe February, or simply “Ekushe”, which is “The 21st” in Bangla, commemorates the day in 1952

when students of the University of Dhaka launched a nationwide protest against the imposition of Urdu on the people of what was then East Pakistan.

• In 1999, in recognition of the Bengali people’s struggle for their language and culture, UNESCO

announced that February 21 would be observed worldwide as International Mother Language Day.

www.YouTube.com/SleepyClasses www.SleepyClasses.com

19

Page 22: February · • The Regulating Act of 1773 created the office with the title of Governor-General of the Presidency of Fort William, or Governor-General of Bengal to be appointed

3. Polity To watch the following questions on YouTube, click on the links given below

• Video 1

• Video 2

1. Which of the following are correct about the Medical Devices (Amendment) Rules, 2020?

1. These are applicable to intended for internal or external use in the diagnosis, treatment, mitigation or prevention of disease or disorder in human beings only

2. It requires  that every medical device, either manufactured in India or imported, will have to have

quality assurance before they can be sold in India

3. The rules are notified in the Drugs and Cosmetics Act by Health Ministry

Select the Correct Option

A. 1 Only

B. 1 and 2 Only

C. 1 and 3 Only

D. All of the above

Answer: D

Explanation:

• The Ministry of Health and Family Welfare recently notified changes in the Medical Devices Rules,

2017 to  regulate medical devices on the same lines as drugs under the Drugs and Cosmetics Act, 1940. Called the Medical Devices (Amendment) Rules, 2020, these are applicable to devices

“intended for internal or external use in the diagnosis, treatment, mitigation or prevention of disease or disorder in human beings or animals” (as notified by the ministry) and require online registration of

these devices “with the Central Licensing Authority through an identified online portal established by

the Central Drugs Standard Control Organisation for this purpose”.

• The rules require that every medical device, either manufactured in India or imported, will have to

have quality assurance before they can be sold anywhere in the country. “After furnishing of the above information on the ‘Online System for Medical Devices’ established by Central Drugs Standard

Control Organisation for this purpose by the applicant’s, registration number will be generated. For

much of the last one year, the health sector has been at the centre of attention following revelations about faulty hip implants marketed by pharma major Johnson & Johnson. This has caused major

embarrassment to the government, too, as it exposed the lack of regulatory teeth when it came to medical devices.

2. Which of the following is correct about the Prohibition of Employment as Manual Scavengers and

their Rehabilitation Act?

1. The law prohibits employing manual scavengers, manual cleaning of sewers and septic tanks

completely

www.YouTube.com/SleepyClasses www.SleepyClasses.com

20

Page 23: February · • The Regulating Act of 1773 created the office with the title of Governor-General of the Presidency of Fort William, or Governor-General of Bengal to be appointed

2. The National Commission for Scheduled Castes issues directions to ensure effective implementation of the law banning manual scavenging

Select the Correct Option

A. 1 only

B. 2 only

C. 1 and 2 only

D. 1 and 3 only

Answer: B

Explanation:

• The Prohibition of Employment as Manual Scavengers and their Rehabilitation (PEMSR) Act came

into force in 2013. The law prohibits employing manual scavengers, manual cleaning of sewers and septic tanks without protective equipment and construction of insanitary latrines. Those violating the

law and getting sewers and septic tanks cleaned without protective equipment can face imprisonment of up to two years or a fine of up to Rs 2 lakh, or both. Repeat offenders will face

imprisonment of up to five years or a fine of up to Rs 5 lakh, or both.

• The Maharashtra government has directed all civic bodies in the state to set up Emergency Response Sanitation Units (ERSUs) to ensure safeguards for sanitation workers who clean manholes and

sewers, after multiple cases were reported of workers dying from suffocation or inhalation of hazardous gases. 

• The Supreme Court had said that making a sanitation worker enter sewer lines without safety gear

should be a crime even in emergency situations. In such instances, if a sanitation worker died due to the unsafe conditions, a compensation of Rs 10 lakh has to be given to the family of the deceased,

stated the court.

• To ensure effective implementation of the law banning manual scavenging, the National commission

for SCs issued various directives. It said workers have to be fully equipped with safety apparatus and

oxygen masks in case they have to clean sewers manually. A first information report has to be lodged against officials or contractors responsible for sending a worker to clean sewers manually, without

proper gear.

3. Which of the following is correct about the Ninth Schedule?

1. The Ninth Schedule contains a list of only central laws which can’t challenged in Courts

2. The schedule was a part of the original Constitution with new amendments leading to addition of laws as required Article 31B can have retrospective operation

Select the Correct Option

A. 1 only

B. 2 only

C. 3 only

www.YouTube.com/SleepyClasses www.SleepyClasses.com

21

Page 24: February · • The Regulating Act of 1773 created the office with the title of Governor-General of the Presidency of Fort William, or Governor-General of Bengal to be appointed

D. 1 and 3 only

Answer: C

Explanation:

• The Ninth Schedule contains a list of central and state laws which cannot be challenged in courts.

Currently, 284 such laws are shielded from judicial review.

• The Schedule became a part of the Constitution in 1951, when the document was amended for the first time. It was created by the new Article 31B, which along with 31A was brought in by the

government to protect laws related to agrarian reform and for abolishing the Zamindari system. While A. 31A extends protection to ‘classes’ of laws, A. 31B shields specific laws or enactments.

• Article 31B also has retrospective operation: meaning if laws are inserted in the Ninth Schedule after

they are declared unconstitutional, they are considered to have been in the Schedule since their commencement, and thus valid. Although Article 31B excludes judicial review, the apex court has said

in the past that even laws under the Ninth Schedule would be open to scrutiny if they violated fundamental rights or the basic structure of the Constitution.

• Recently, LJP leader Chirag Paswan said that reservation should be put under the Ninth Schedule of

the Constitution. His comments came days after the Supreme Court ruled that reservation in the matter of promotions in public posts was  not a fundamental right, and that a state cannot be

compelled to offer quota if it chooses not to.

4. Which of the following is correct about the Municipal Performance Index (MPI) 2019?

1. The Municipal Performance Index has been launched by the NITI Aayog

2. The Index is designed to assess quality of life of citizens in 100 Smart Cities only

Select the Correct Option

A. 1 only

B. 2 only

C. Both 1 and 2

D. Neither 1 nor 2

Answer: D

Explanation:

• To help assess the progress made in cities through various initiatives and empower them to use

evidence to plan, implement & monitor their performance, two Assessment Frameworks, viz. Ease of

LivingIndex (EoLI) and Municipal Performance Index (MPI) 2019 have been launched by the Ministry of Housing & Urban Affairs. Both these indices are designed to assess quality of life of

citizens in 100 Smart Cities and 14 other Million Plus Cities. 

www.YouTube.com/SleepyClasses www.SleepyClasses.com

22

Page 25: February · • The Regulating Act of 1773 created the office with the title of Governor-General of the Presidency of Fort William, or Governor-General of Bengal to be appointed

• With the Municipal Performance Index 2019, the Ministry has sought to assess the performance of municipalities based on five enablers namely Service, Finance, Planning, Technology and Governance

which have been further divided into 20 sectors which will be evaluated across 100 indicators. This will help Municipalities in better planning and management, filling the gaps in city administration, and

improving the liveability of cities for its citizens.

5. Which of the following are correct about the Emergency Response Support System (ERSS)?

1. The ERSS is being implemented by Ministry of Home Affairs across all states and UTs

2. The ERSS provides a single response number for women stuck is situations of distress, panic, exploitation or violence in public or private places

Select the Correct Option

A. 1 only

B. 2 only

C. Both 1 and 2

D. Neither 1 nor 2

Answer: A

Explanation:

The Ministry of Home Affairs is implementing Emergency Response Support System (ERSS) project

through States/Union Territories. ERSS seeks to enable a pan-India; single, internationally recognized number i.e. 112 based emergency response mechanism for various emergencies, including police,

medical and fire, with computer-aided dispatch of field resources. Addition of other emergency

services is a function of requirement and preparedness of States/ Union Territories. 112 service is accessible through call, SMS, e-mail, panic button and a 112 India mobile app.

6. Which of the following is correct with regard to Right to establish educational institutions under Article 30?

1. Article 30 provides all minorities shall have the right to establish and administer educational

institutions of their choice

2. The right conferred on minorities under Article 30 is only to ensure equality with the majority and

not intended to place the minorities in a more advantageous position

3. No conditions can be imposed by the State to ensure proper utilization of the aid provided to

minority institutions

Select the Correct Option

A. 1 only

B. 1 and 2 only

C. 1 and 3 only

D. 2 and 3 only

www.YouTube.com/SleepyClasses www.SleepyClasses.com

23

Page 26: February · • The Regulating Act of 1773 created the office with the title of Governor-General of the Presidency of Fort William, or Governor-General of Bengal to be appointed

Answer: B

Explanation:

Article 30 grants the following rights to minorities, whether religious or linguistic:

1. All minorities shall have the right to establish and administer educational institutions of their

choice.

2. The compensation amount fixed by the State for the compulsory acquisition of any property of a minority educational institution shall not restrict or abrogate the right guaranteed to them. This

provision was added by the 44th Amendment Act of 1978 to protect the right of minorities in this regard. The Act deleted the right to property as a Fundamental Right (Article 31).

3. In granting aid, the State shall not discriminate against any educational institution managed by a

minority. In a judgement delivered in the Secretary of Malankara Syrian Catholic College case (2007), the Supreme Court has summarized the general principles relating to establishment and

administration of minority educational institutions.

• The right conferred on minorities under Article 30 is only to ensure equality with the majority and not

intended to place the minorities in a more advantageous position vis-à-vis the majority. There is no

reverse discrimination in favour of minorities.

• The general laws of the land relating to national interest, national security, social welfare, public

order, morality, health, sanitation, taxation etc., applicable to all, will equally apply to minority institutions also. Extention of aid by the State, does not alter the nature and character of the minority

educational institutions.

• The conditions can be imposed by the State to ensure proper utilization of the aid, without however diluting or abridging the right under Article 30(1).

7. Which of the following is regarded as ‘information’ under the RTI Act?

1. Software installed in government apps

2. Logbooks of a government department

3. Electronic Voting machine

4. Service details of a government servant

Select the Correct Option

A. 2 only

B. 1 and 2 only

C. 2 and 3 only

D. 1,2 and 4 only

Answer: C

www.YouTube.com/SleepyClasses www.SleepyClasses.com

24

Page 27: February · • The Regulating Act of 1773 created the office with the title of Governor-General of the Presidency of Fort William, or Governor-General of Bengal to be appointed

Explanation:

Article 19 (1) of the Indian Constitution guarantees us the right to freedom of speech and expression

and as recognized by the Supreme Court this also implies a full right to information. The Right to Information Act, 2005 (“the Act”) has established the necessary practical regime of right to information.

Under this Act, an Indian Citizen can access the information from the public authority. Right to

Information means the Right to Information accessible under this Act which is held by or under the possession of any Central or State Public Authority and that includes the right to:

• Inspection of work, documents, records;

• Taking notes, extracts or certified copies of documents or records;

• Taking certified samples of material;

• Obtaining information in the form of diskettes, floppies, tapes, video cassettes or in any other electronic mode or through printouts where such information is stored in a computer or in any other

device.

Section 8(1)(d) exempts information including commercial confidence, trade secrets or intellectual

property, the disclosure of which would harm the competitive position of a third party unless the

competent authority is satisfied that larger public interest warrants the disclosure of such information.

Section 8(1)(j) of RTI Act,2005 states as ”Notwithstanding anything contained in this act , there shall be

no obligation to give any citizen information which relates to personal information the disclosure of which has no relationship to any public activity or interest , or which would cause unwarranted invasion

of the privacy of the individual unless the Central Public Information Officer or the State Public

Information Officer or the appellate authority, as the case may be, is satisfied that the larger public interest justifies the disclosure of such information.”

• In 2017, the Supreme Court has held that service details of employees fall within the ambit of ‘personal information’ under Section 8(1)(j) of the Right to Information Act and that such details

cannot be furnished unless any nexus with the larger public interest is shown.

• In 2019, the Central Information Commission (CIC) has ruled that the EVM machine is information under the Right to Information Act and directed the Election Commission to respond to the appellant.

• The ECI though contended that the software installed in the EVM is an intellectual property of a third party, the disclosure of which would harm the competitive position of the third party concerned.

8. Which of the following is/are correct about Delhi’s Happiness Curriculum?

1. Inspired by the World Happiness Report the curriculum has been included in government and private schools Delhi

2. There is no evaluation or marks regarding the happiness curriculum and only a qualitative assessment is included

Select the Correct Option

A. 1 only

B. 2 only

www.YouTube.com/SleepyClasses www.SleepyClasses.com

25

Page 28: February · • The Regulating Act of 1773 created the office with the title of Governor-General of the Presidency of Fort William, or Governor-General of Bengal to be appointed

C. Both 1 and 2

D. Neither 1 nor 2

Answer: B

Explanation:

• The happiness curriculum is one of the flagship schemes of the Delhi government in the education

sector launched in July 2018 in all government schools. Citing the World Happiness Report, 2018, in which India ranked 133 among 155 nations in the global rankings, the curriculum calls for schools in

India to promote development in cognition, language, literacy, numeracy and the arts along with addressing the well-being and happiness of students.

• The objectives of this curriculum include developing self-awareness and mindfulness, inculcating

skills of critical thinking and inquiry, enabling learners to communicate effectively and helping learners to apply life skills to deal with stressful and conflicting situations around them.

• The assessment under this curriculum is qualitative with no marks or evaluation, focusing on the “process rather than the outcome” and noting that each student’s journey is unique and different.

9. Which of the following is/ are correct regarding maintenance of public order?

1. In circumstances where public disorder is not so serious as to fall in the category of an “internal disturbance” as defined in Article 355 of the Constitution army can’t be requested

2. The Seventh Schedule of the Constitution states that use of the armed forces in the maintenance of public order is within the purview of the states

Select the Correct Option

A. 1 only

B. 2 only

C. Both 1 and 2

D. Neither 1 nor 2

Answer: D

Explanation:

• While public order and police are under the state list, the state government may request the Union

government to make available armed forces to help restore public order.

• Even in circumstances where public disorder is not so serious as to fall in the category of an “internal

disturbance” as defined in Article 355 of the Constitution, the Union Government may accede to the

request.

• However, it is important to note that except for the limited purpose of dispersing an “unlawful

assembly” and arresting its members — for which Section 130 CRPC empowers the Executive Magistrate to requisition the aid of the Army — neither the state government nor any authority under

it has been conferred by the Constitution any legal right to call the armed forces while dealing with a

public disorder or “internal disturbance”.

www.YouTube.com/SleepyClasses www.SleepyClasses.com

26

Page 29: February · • The Regulating Act of 1773 created the office with the title of Governor-General of the Presidency of Fort William, or Governor-General of Bengal to be appointed

• Also, the Seventh Schedule of the Constitution — which deals with the subject of public order in the state list — states that use of the armed forces in the maintenance of public order is outside the

purview of the states.

10.Which of the following statements are correct?

1. The origin of the Indian railway budget was the result of a report by British politician William

Ackworth in 1924

2. While the Union budget is a Constitutional requirement, there is no such necessity attached with

the Railways budget

Select the Correct Option

A. 1 only

B. 2 only

C. Both 1 and 2

D. Neither 1 nor 2

Answer: C

Explanation:

• The origin of the railway budget goes back to a report by British politician William Ackworth in 1924. He recommended a separate railway budget, given that most of the infrastructure spending by the

British government went towards building railway lines.

• While the Union budget is a Constitutional requirement and is presented under Article 112 of the

Indian Constitution, which mandates an annual financial statement, the Constitution does not talk

about the railway budget in particular

www.YouTube.com/SleepyClasses www.SleepyClasses.com

27

Page 30: February · • The Regulating Act of 1773 created the office with the title of Governor-General of the Presidency of Fort William, or Governor-General of Bengal to be appointed

4. International Relations The video links to watch the following questions on YouTube are provided in the Science & Tech Section

1. Which of the following South American country is landlocked?

A. Uruguay

B. Paraguay

C. Ecuador

D. Both A and B

Answer: B

Explanation:

2. Which of the following statement(s) in the context of Nile river are correct

1. The longest African river runs through 11 countries

2. Grand Ethiopian Renaissance Dam is being constructed on the White Nile river

Select the Correct Option

1. 1 only

2. 2 only

3. Both 1 and 2

4. Neither 1 nor 2

www.YouTube.com/SleepyClasses www.SleepyClasses.com

28

Page 31: February · • The Regulating Act of 1773 created the office with the title of Governor-General of the Presidency of Fort William, or Governor-General of Bengal to be appointed

Answer: A

Explanation:

• Africa’s longest river, the Nile runs through 11 countries. One of them is Ethiopia which contributes about 85% of Nile water flowing to Sudan and Egypt

• Grand Ethiopian Renaissance Dam or Millennium Dam or Hidase Dam is a gravity dam on the Blue

Nile river in Ethiopia

3. Azores and Madeira islands located in the Atlantic ocean are under the sovereignty of which of the following countries?

A. Spain

B. United Kingdom

C. Portugal

D. France

Answer: C

4. Barakah nuclear power plant is the first nuclear plant in the Arab world. It is located in which of the following countries?

A. United Arab Emirates

B. Saudi Arabia

www.YouTube.com/SleepyClasses www.SleepyClasses.com

29

Page 32: February · • The Regulating Act of 1773 created the office with the title of Governor-General of the Presidency of Fort William, or Governor-General of Bengal to be appointed

C. Iraq

D. Egypt

Answer: A

Explanation:

• United Arab Emirates took a final step toward switching on the Arab world’s first commercial nuclear

power plant

• It has now entered in the elite club of 30 countries that make power from nuclear energy

• It is built and run by a joint venture with Korea Electric Power Corp.

• Barakah is the first of four civilian reactors that the government plans to fire up by 2023

• The plants, located on a sparsely populated strip of desert on the Persian Gulf coast, are estimated to

cost $25 billion

5. Which of the following separates Great Britain from continental Europe?

A. Strait of Dover

B. North Channel

C. Davis Strait

D. Skagerrak Strait

Answer: A

Explanation:

• North Channel or Irish Channel is the strait between north-eastern Northern Ireland and south-western Scotland

• Davis Strait is a northern arm of the Labrador Sea. It lies between mid- western Greenland and

Canada

www.YouTube.com/SleepyClasses www.SleepyClasses.com

30

Page 33: February · • The Regulating Act of 1773 created the office with the title of Governor-General of the Presidency of Fort William, or Governor-General of Bengal to be appointed

6. Scientists have proposed Northern European Enclosure Dam (NEED) enclosing all of the North Sea. Dams would be constructed between

A. Scotland and Norway

B. France and England

C. Both A and B

D. Neither A nor B

Answer: C

Explanation:

• Scientists have suggested building two enclose the North Sea and defend northern Europe from rising

sea levels

• Called the Northern European Enclosure Dam (NEED), the proposal suggests building dams in the English Channel between France and England and between Scotland and Norway

www.YouTube.com/SleepyClasses www.SleepyClasses.com

31

Page 34: February · • The Regulating Act of 1773 created the office with the title of Governor-General of the Presidency of Fort William, or Governor-General of Bengal to be appointed

5. Economy To watch the following questions on YouTube, click on the links given below

• Video 1

• Video 2

• Video 3

1. It has been announced in Budget that, Fibre to the Home (FTTH) connections through Bharatnet will link 100,000 gram panchayats this year.

Consider the following in context of the Optical Fibre used to create the National Optical Fiber

Network (NOFN) in India:

1. Fiber-optic communication is a method of transmitting information from one place to another by

sending pulses of infrared light.

2. Electrical cabling is preferred over Fiber when high bandwidth, long distance, or immunity to

electromagnetic interference are required.

Select the Correct Option

A. 1 Only

B. 2 only

C. Both are Correct

D. None is Correct

Answer: A

Explanation:

• Fiber-optic communication is a method of transmitting information from one place to another by sending pulses of infrared light through an optical fiber.

• The light forms an electromagnetic carrier wave that is modulated to carry information.

• Fiber is preferred over electrical cabling when high bandwidth, long distance, or immunity to electromagnetic interference are required.

• This type of communication can transmit voice, video, and telemetry through local area networks, computer networks, or across long distances.

• Optical fiber is used by many telecommunications companies to transmit telephone signals, Internet

communication, and cable television signals.

• The National Optical Fibre Network (NOFN) aims to connect all the 2,50,000 Gram panchayats in the

country and provide 100 Mbps connectivity to all gram panchayats (GPs).

• The NOFN project was funded by the Universal Service Obligation Fund (USOF).

• Based on NOFN experiences, newer, updated and upgraded version – Bharat Net was conceived as a

nation-wide broadband network.

www.YouTube.com/SleepyClasses www.SleepyClasses.com

32

Page 35: February · • The Regulating Act of 1773 created the office with the title of Governor-General of the Presidency of Fort William, or Governor-General of Bengal to be appointed

• Bharat Net is a project of national importance to establish, by 2017, a highly scalable network infrastructure accessible on a non-discriminatory basis, to provide on demand, affordable broadband

connectivity of 2 Mbps to 20 Mbps for all households and on demand capacity to all institutions, to realise the vision of Digital India, in partnership with States and the private sector.

2. It has been announced in the budget that 5 archaeological sites will be developed as iconic sites with

on-site Museums. Which of the following sites are correctly matched with their respective states?

1. Rakhigarhi – Haryana

2. Hastinapur - Uttar Pradesh

3. Shivsagar – Karnataka

4. Dholavira – Rajasthan

5. Adichanallur – Kerala

Select the Correct Option

A. 1 Only

B. 1 & 2 Only

C. 4 & 5 only

D. All are Correct

Answer: B

Explanation:

These 5 archaeological sites will be developed as iconic sites with on-site Museums.

1. Rakhigarhi – Haryana

2. Hastinapur - Uttar Pradesh

3. Shivsagar – Assam

4. Dholavira – Gujarat

5. Adichanallur - Tamil Nadu

There’s also a proposal to set-up a Maritime museum to be set up at Lothal- the Harrapan age maritime

site near Ahmedabad, by Ministry of Shipping.

3. A sum of Rs. 100 crore has been allocated to begin the preparations for G20 presidency to be hosted

in India. India will host the G20 summit in the year:

A. 2020

B. 2021

C. 2022

D. 2023

Answer: C www.YouTube.com/SleepyClasses

www.SleepyClasses.com 33

Page 36: February · • The Regulating Act of 1773 created the office with the title of Governor-General of the Presidency of Fort William, or Governor-General of Bengal to be appointed

Explanation

2022 - which will be India's 75th anniversary as an independent nation.

2020 – Saudi Arabia

2021 - Italy

• The G20 is an international forum for the governments and central bank governors from 19 countries

and the European Union.

• It originated in 1999 at the level of Finance Ministers and Central Bank Governors.

• In the wake of the 2008 global financial crisis, the G20 was elevated to include the leaders of member countries.

• The first G20 Leaders’ Summit took place in Washington D.C. in November 2008.

• The Group of 20 nations (G20) is an international grouping consisting of: Argentina, Australia, Brazil, Canada, China, the European Union, France, Germany, India, Indonesia, Italy, Japan, Mexico, Russia,

Saudi Arabia, South Africa, South Korea, Turkey, the UK and the US.

4. It has been announced in the budget that Deposit Insurance and Credit Guarantee Corporation

(DICGC) is permitted to increase Deposit Insurance Coverage to Rs. 5 lakh from Rs.1 lakh per

depositor. In context of DICGC, consider the following:

1. Regional Rural Banks are exempt from the purview of DICGC.

2. DICGC does not insure deposits of Central/State Governments

Select the Correct Option

A. 1 only

B. 2 only

C. Both are correct

D. None is correct

Answer: B

Explanation: The following are required to be registered under DICGC act:

1. All commercial banks

2. Regional Rural Banks

3. Co-operative Banks

4. A primary co-operative credit society becoming a primary co-operative bank

DICGC insures all bank deposits, such as saving, fixed, current, recurring, etc.

Except

The following types of deposits:

www.YouTube.com/SleepyClasses www.SleepyClasses.com

34

Page 37: February · • The Regulating Act of 1773 created the office with the title of Governor-General of the Presidency of Fort William, or Governor-General of Bengal to be appointed

• Deposits of foreign Governments

• Deposits of Central/State Governments

• Inter-bank deposits

• Deposits of the State Land Development Banks with the State co-operative banks

• Any amount due on account of and deposit received outside India

• Any amount which has been specifically exempted by the corporation with the previous approval of the RBI.

5. Dividend Distribution Tax (DDT) is a:

A. Direct Tax

B. Indirect Tax

C. Both A & B

D. Neither A nor B

Answer: A

Explanation:

• A dividend is a return given by a company to its shareholders out of the profits earned by the

company in a particular year.

• Dividend constitutes income in the hands of the shareholders.

• Before the budget announcement,

• The Dividend Distribution Tax, or DDT, used to be taxable at source, and was deducted at the time of

the company distributing dividends.

✓Dividends will now be taxed in the hands of recipients at their applicable rate.

6. Which of the following is/are correct in context of AGR (Adjusted Gross Revenue)?

A. It is a part of profit-sharing model announced in the National Telecom Policy, 1999.

B. It is a part of revenue-sharing model announced in the National Telecom Policy, 1999.

C. It is a part of a fixed annual licence fee announced in the National Telecom Policy, 1999.

D. None of the above

Answer: B

Explanation: Why in News

www.YouTube.com/SleepyClasses www.SleepyClasses.com

35

Page 38: February · • The Regulating Act of 1773 created the office with the title of Governor-General of the Presidency of Fort William, or Governor-General of Bengal to be appointed

7. Pandemic bonds were launched by:

A. The World Bank

B. World Health Organisation

C. Asian Development Bank

D. None of the above

Answer: A

Explanation

The Ebola outbreak inspired the World Bank to develop a so-called “pandemic catastrophe bond,”.

They were an instrument designed to quickly provide financial support in the event of an outbreak.

What are Catastrophe Bonds?

• They are a way to transfer risk, often for natural disasters.

• If a specific qualifying event occurs, such as if claims from a natural disaster exceed a certain amount,

the bond holders forfeit the principal of the bond, which goes to the insurer to help defray costs.

• Pandemic bonds are triggered by, for example, the number of patients or the speed of disease spread

(a “parametric trigger”).

World Bank’s Pandemic Bonds

• The World Bank announced the creation of the PEF in May 2016 at the G7 Finance Ministers and

Central Governors meeting in Sendai, Japan.

• The PEF will quickly channel funding to countries facing a major disease outbreak with pandemic

potential.

• Its unique financing structure combines funding from the bonds issued today with over-the-counter derivatives that transfer pandemic outbreak risk to derivative counterparties.

• The bonds are issued by the World Bank's International Bank for Reconstruction and Development (IBRD).

• The bonds offer investors high coupons in return for the risk of having to forgo some or all their

money in the event of pandemic outbreaks of a number of infectious diseases.

8. In context of ESOP (Employee stock ownership plan) which of the following is/are correct:

1. They help companies to retain employees for a long-term.

2. ESOPs are not yet allowed in India.

Select the Correct Option

A. 1 only

B. 2 only

C. Both are correct

www.YouTube.com/SleepyClasses www.SleepyClasses.com

36

Page 39: February · • The Regulating Act of 1773 created the office with the title of Governor-General of the Presidency of Fort William, or Governor-General of Bengal to be appointed

D. None is correct

Answer: A

Explanation: In News:

The Finance Minister has assured start-ups to resolve ESOP tax related issue

What are ESOPs

• ESOPs enable employees to buy the company’s shares at a discounted price.

• Companies and, usually, startups roll out this scheme for selected employees, based on their position

and ability to impact the company.

• This ownership comes in the form of company shares, which is assured upon fulfilling predefined

conditions.

• But most often, ESOPs become a part of their compensation offering in startups, to motivate employees to give their best at work.

• ESOPs give the right to directors, officers and employees of the company to subscribe to the company share at a future date at a predetermined rate.

• Companies registered as private limited can benefit from this by following the Companies Act, 2013

and rules made thereon.

• Employees who opt to convert the ESOP to shares will have those shares taxable under the Income

Tax Act, 1961.

9. In context of The Essential Commodities Act (ECA) consider the following:

1. The act is applicable on Grains, Pulses and Fertilisers only.

2. The Centre can include new commodities as and when the need arises without waiting for Parliament’s approval.

Select the Correct Option

A. 1 only

B. 2 only

C. Both are correct

D. None is correct

Answer: B

Explanation:

In News

Because of Coronavirus outbreak, Provisions of Essential Commodities Act may be invoked to ensure adequate supply of drugs.

What is ECA?

www.YouTube.com/SleepyClasses www.SleepyClasses.com

37

Page 40: February · • The Regulating Act of 1773 created the office with the title of Governor-General of the Presidency of Fort William, or Governor-General of Bengal to be appointed

• Enacted in 1955.Used by the Government to regulate the production, supply and distribution of commodities declared ‘essential’.

• This is done in order to make them available to consumers at fair prices.

What all does it include?

• The list of items under the Act include drugs, fertilisers, pulses and edible oils, and petroleum and

petroleum products.

• The Centre can include new commodities as and when the need arises, and take them off the list once

the situation improves.

• Legal Metrology (Packaged Commodities) Rules 2011 is linked to the ECA.

• The Government can fix the retail price of any packaged commodity that falls under the ECA.

10.Totalizer Machines were in news recently. The Machines are related to:

A. Cricket

B. Artificial Intelligence

C. Elections

D. Machine learning

Answer: C

Explanation:

• Simply put, a totaliser is a mechanism which allows votes from 14 booths to be counted together so that voters are saved from pre-poll intimidation and post-poll harassment.

• Currently, the votes cast via EVMs are counted on individual booth basis. The interface connects to

the main control units of a 14 EVM cluster and the consolidated vote count for candidates is obtained merely by pressing the result button on the totaliser.

• This result obtained is without disclosure of votes polled by candidates at particular voting booths/stations.

• Before the introduction of EVMs, ballot papers were often mixed to prevent intimidation of voters by

disclosure of voting pattern.

11.Which of the following is/are true about FOREX?

1. Only US Dollars are considered as Foreign Currency Assets (FCAs)

2. In terms of assets with IMF, only Special Drawing Rights are considered (and not RBI’s reserve

position)

Select the Correct Option

A. 1 only

B. 2 only

C. Both are correct www.YouTube.com/SleepyClasses

www.SleepyClasses.com 38

Page 41: February · • The Regulating Act of 1773 created the office with the title of Governor-General of the Presidency of Fort William, or Governor-General of Bengal to be appointed

D. None is correct

Answer: D

www.YouTube.com/SleepyClasses www.SleepyClasses.com

39

Page 42: February · • The Regulating Act of 1773 created the office with the title of Governor-General of the Presidency of Fort William, or Governor-General of Bengal to be appointed

Explanation:

12.Which of the following is correct in context of investment in Sovereign Gold Bonds?

1. They are issued by Reserve Bank of India.

2. The minimum investment in these bonds is one gram

Select the Correct Option

A. 1 only

B. 2 only

C. Both are correct

D. None of the above

Answer: C

Explanation: Sovereign Gold Bond Scheme

What is Sovereign Gold Bond (SGB)?

• SGBs are government securities denominated in grams of gold. They are substitutes for holding

physical gold.

Who is the issuer?

• The Bond is issued by Reserve Bank on behalf of Government of India.

What is the minimum and maximum limit for investment?

• The Bonds are issued in denominations of one gram of gold and in multiples thereof.

• Minimum investment in the Bond shall be one gram with a maximum limit of subscription of 4 kg for

individuals.

What will I get on redemption?

• On maturity, the redemption price shall be based on simple average of closing price of gold of 999 purity of previous 3 business days from the date of repayment, published by the India Bullion and

Jewelers Association Limited

www.YouTube.com/SleepyClasses www.SleepyClasses.com

40

Page 43: February · • The Regulating Act of 1773 created the office with the title of Governor-General of the Presidency of Fort William, or Governor-General of Bengal to be appointed

13.Which of the following is/are correct in context of Financial Action Task Force (on Money Laundering):

1. It was founded in 1989 on an OECD initiative.

2. FATF grey lists a country which it considers as a safe haven for terror funding and money

laundering.

3. Pakistan will be the first country ever to be blacklisted.

Select the Correct Option

A. 2 only

B. 1 & 2 only

C. 3 only

D. All are correct

Answer: A

Explanation:

It was founded in 1989 on a G-7 initiative. So far, only two countries have been blacklisted, they are Iran

and North Korea.

14.In context of SAARC Currency Swap Facility, consider the following statements:

1. SAARC Currency Swap Facility came into operation in 2012.

2. The present swap arrangement is of overall corpus of US $10 billion.

3. The drawals can be made in US Dollar, Euro or Indian Rupee.

Which of the above is/are correct?

A. 1 & 2 only

B. 2 & 3 only

C. 1 & 3 only

D. All of the above

Answer: C

Explanation: SAARC Currency Swap Facility 

• To further financial stability and economic cooperation within the SAARC region, the Reserve Bank of

India, with the concurrence of the Government of India, has decided to put in place a revised Framework on Currency Swap Arrangement for SAARC countries 2019-2022.

• The Framework is valid from November 14, 2019 to November 13, 2022.

• Based on the terms and conditions of the Framework, the RBI would enter into bilateral swap agreements with SAARC central banks, who want to avail swap facility.

www.YouTube.com/SleepyClasses www.SleepyClasses.com

41

Page 44: February · • The Regulating Act of 1773 created the office with the title of Governor-General of the Presidency of Fort William, or Governor-General of Bengal to be appointed

• It may be recalled that the SAARC Currency Swap Facility came into operation on November 15, 2012 with an intention to provide a backstop line of funding for short term foreign exchange liquidity

requirements or balance of payment crises till longer term arrangements are made.

• Under the Framework for 2019-22, RBI will continue to offer swap arrangement within the overall

corpus of US $ Two billion.

• The drawals can be made in US Dollar, Euro or Indian Rupee.

• The Framework provides certain concessions for swap drawals in Indian Rupee.

• The Currency Swap Facility will be available to all SAARC member countries, subject to their signing the bilateral swap agreements.

15.Codex Trust Fund is run under the aegis of:

A. WHO

B. FAO

C. Both of the above

D. None of the above

Answer: C

www.YouTube.com/SleepyClasses www.SleepyClasses.com

42

Page 45: February · • The Regulating Act of 1773 created the office with the title of Governor-General of the Presidency of Fort William, or Governor-General of Bengal to be appointed

6. Environment To watch the following questions on YouTube, click on the links given below

• Video 1

• Video 2

• Video 3

1. What is meant by stranded carbon?

A. It refers to fossil fuel energy resources that will never be burnt if the world is to adhere to a given

carbon budget.

B. It refers to fossil fuel energy resources that have been burnt and are present in the atmosphere.

C. The carbon dioxide that has been geo-engineered back into the Earth.

D. None of the above

Answer: A

Explanation:

• Stranded Carbon refers to fossil fuel energy resources that cannot be burnt if the world is to adhere to a given carbon budget.

• Therefore some of proven reserves of fossil fuels will never be burnt and will remain stranded

2. Which of the following pairs is/are not matched correctly?

1. Vembanad lake – Kerala

2. Pulicat lake – Andhra Pradesh and Tamil Nadu

3. Chilika lake – Andhra Pradesh

Select the correct answer using codes given below

A. 1 and 3 only

B. 2 and 3 only

C. 3 only

D. All of the above

Answer: C

Explanation:

• Vembanad (Vembanad Kayal or Vembanad Kol) is the longest lake in India, and the largest lake in the

state of Kerala.

• The Vembanad Wetland system was included in the list of wetlands of international importance, as

defined by the Ramsar Convention for the conservation and sustainable utilization of wetlands in 2002. 

www.YouTube.com/SleepyClasses www.SleepyClasses.com

43

Page 46: February · • The Regulating Act of 1773 created the office with the title of Governor-General of the Presidency of Fort William, or Governor-General of Bengal to be appointed

• Nestled on the border of Andhra Pradesh and Tamil Nadu, Pulicat Lake is India's second largest lagoon. Although 90% of the lake falls in Andhra Pradesh, the other 10% comes under Protected

Areas of Tamil Nadu.

• Chilika Lake  is a brackish water  lagoon in Odisha  state on the east coast of  India, at the mouth of

the Daya River, flowing into the Bay of Bengal, covering an area of over 1,100 km.

• It is the largest coastal lagoon in India and the second largest brackish water lagoon in the world[after The New Caledonian barrier reef.

• In 1981, Chilika Lake was designated the first Indian  wetland of international importance  under the Ramsar Convention.

3. Consider the following statements regarding Leopards

1. The IUCN status is Endangered.

2. Like the tiger census held once in four years, there is a dedicated census for the leopard also.

Which of the above statements is/are correct?

A. 1 only

B. 2 only

C. Both 1 and 2

D. Neither 1 nor 2

Answer: D

Explanation:

• Unlike the tiger census held once in four years, there is no dedicated census for the leopard.

• However, a leopard count accompanying the tiger census of 2014 estimated its population to be 12,000-14,000 with about 8,000 in the vicinity of tiger habitat.

• The leopard, probably the most widely distributed and highly adaptable large felid (a member of the cat family) globally, is still persisting in most of its historic range, but there is a lack of sub-species-

level population data on country or regional scale.

4. Consider the following statements regarding phumdi

1. It is an floating assortment of soil, vegetation and organic matter in various stages of decay.

2. The phumdi’s life cycle is regulated by the seasonal fluctuation in water level. 

3. It is found in Manipur.

www.YouTube.com/SleepyClasses www.SleepyClasses.com

44

Page 47: February · • The Regulating Act of 1773 created the office with the title of Governor-General of the Presidency of Fort William, or Governor-General of Bengal to be appointed

Which of the above statement is correct?

A. 3 only

B. 1 and 3 only

C. 2 and 3 only

D. All of the above

Answer: D

Explanation:

• Phumdis  are a series of  floating islands, exclusive to the  Loktak Lake  in  Manipur  state, in northeastern India. 

• A floating assortment of soil, vegetation and organic matter in various stages of decay,

the phumdi clump together to form islands that move around freely on the lake, their shape and size morphing through the year. 

•  Like any living organism, the phumdi’s life cycle is regulated by the seasonal fluctuation in water level.

• In the dry season, they sink to the lake bed where their roots absorb nutrients from the soil.

• During monsoons they float back to the surface.

5. Consider the following

1. Epiphytic 

2. Terrestrial 

3. Mycoheterotrophic

Which of the above are the different types of orchids?

A. 1 and 2 only

B. 1 only

C. 2 only

D. All of the above

Answer: D

Explanation:

• Orchids can be broadly categorised into three life forms:

✓Epiphytic (plants growing on another plants including those growing on rock boulders and often termed lithophyte), Terrestrial (plants growing on land and climbers) and Mycoheterotrophic

(plants which derive nutrients from mycorrhizal fungi that are attached to the roots of a vascular

plant).

• About 60% of all orchids found in the country, which is 757 species, are epiphytic, 447 are terrestrial

and 43 are mycoheterotrophic.

www.YouTube.com/SleepyClasses www.SleepyClasses.com

45

Page 48: February · • The Regulating Act of 1773 created the office with the title of Governor-General of the Presidency of Fort William, or Governor-General of Bengal to be appointed

• The epiphytic orchids are abundant up to 1800 m above the sea level and their occurrence decreases with the increase in altitude.

• Terrestrial orchids, which grow directly on soil, are found in large numbers in temperate and alpine region whereas mycoheterotrophic orchids, mostly associated with ectomycorrhizal fungi, are found

in temperate regions, or are found growing with parasites in tropical regions.

• A State-wise distribution of orchid species point out that the Himalayas, North-East parts of the country and Western Ghats are the hot-spots of the beautiful plant species.

• The highest number of orchid species is recorded from Arunachal Pradesh with 612 species, followed by Sikkim 560 species and West Bengal.

• Another interesting fact is that the entire orchid family is listed under appendix II of CITES

(Convention on International Trade in Endangered Species of Wild Fauna and Flora) and hence any trade of wild orchid is banned globally.

6. Consider the following statements regarding Future of Earth report 2020

1. It has been released by the South Asia Future Earth Regional Office, Divecha Centre for Climate

Change, Indian Institute of Science.

2. The aim of the report is reducing carbon footprint and halting global warming below 2 degree Celsius by 2100.

Which of the above are correct?

A. 1 only

B. 2 only

C. Both 1 and 2

D. Neither 1 nor 2

Answer: A

Explanation:

• Five global risks that have the potential to impact and amplify one another in ways that may cascade

to create global systemic crisis, have been listed by “The Future of Earth, 2020”, which was released by the South Asia Future Earth Regional Office, Divecha Centre for Climate Change, Indian Institute

of Science.

• The report lists failure of climate change mitigation and adaptation; extreme weather events; major

biodiversity loss and ecosystem collapse; food crises; and water crises, as the five global risks.

• The report was prepared with the aim of reducing carbon footprint and halting global warming below 2 degree Celsius by 2050.

7. Consider the following statements regarding Nilgiri Tahr

1. The IUCN status is critically endangered.

2. It has been listed under Schedule 1 of the Wildlife (Protection) Act, 1972.

www.YouTube.com/SleepyClasses www.SleepyClasses.com

46

Page 49: February · • The Regulating Act of 1773 created the office with the title of Governor-General of the Presidency of Fort William, or Governor-General of Bengal to be appointed

3. Mukurthi National Park has been created to protect Nilgiri Tahr.

4. It is endemic to the Nilgiri Hills.

Which of the above are correct?

A. 1 and 4 only

B. 1, 2 and 4 only

C. 2, 3 and 4 only

D. All of the above

Answer: C

Explanation:

• Nilgiri tahr is the state animal of Tamil Nadu. It has been listed as “Endangered” by IUCN.

• It has been listed under Schedule 1 of the Wildlife (Protection) Act, 1972.

• It is endemic to Nilgiris.

• It is found in open montane grassland habitat of rain forests ecoregion.

• Mukurthi National Park  (MNP) is a protected area  located in the western corner of

the Nilgiris Plateau west of Ootacamund hill station  in the northwest corner of Tamil Nadu state in

the Western Ghats mountain range of South India.

• The park was created to protect its keystone species, the Nilgiri tahr.

• It is a part of Nilgiri Biosphere Reserve, India's first International Biosphere Reserve.

8. Consider the following statements regarding saltwater crocodiles

1. They are also known as estuarine crocodiles.

2. They are included in Schedule I of Wildlife Protection Act.

3. Their IUCN Status is endangered.

Which of the above statements is/are correct?

A. 1 and 2 only

B. 2 and 3 only

C. 1 and 3 only

D. All of the above

Answer: A

Explanation:

• Saltwater crocodiles, known as ‘salties’, are native to eastern India, South-East Asia and Northern

Australia.

www.YouTube.com/SleepyClasses www.SleepyClasses.com

47

Page 50: February · • The Regulating Act of 1773 created the office with the title of Governor-General of the Presidency of Fort William, or Governor-General of Bengal to be appointed

• Before conservation efforts started in these countries in the 1960s, they were often poached for their skin, meat and as trophy kills.

• Also called estuarine crocodile is found in Inland lakes, swamps and marshes as well as coastal brackish waters and tidal sections of rivers.

• The Indian government launched its conservation effort, Project Crocodile, in 1975. Saltwater

crocodiles, like tigers and rhinoceroses, were given ‘Schedule 1’ protection under the new Wildlife Protection Act.

• Their killing was punishable up to seven years in prison.

• Found in coastal areas of Odisha, WB and TN.

• Their status is Least concern in IUCN Red List.

• Citing the growing number of crocodile attacks on tourists, the administration of Andaman and Nicobar islands had last year requested the central government to delist the saltwater crocodiles (or

salties as they are referred to) from Schedule 1 of the Wildlife Protection Act, 1972.

• This was followed with the proposal of culling the animals to control their growing population on the

island.

• The rationale provided was that the rise in their numbers was leading to an increase in instances of human-crocodile conflicts.

• This is when the environment ministry roped in Doon-based Wildlife Institute of India (WII) to check the feasibly of the request.

• After conducting an over six-month-long survey, WII observed that the situation did not warrant de-

notification of the salties and neither was culling a solution.

9. Apiary on Wheels’ is under which of the following ministry?

A. Ministry of Agriculture and Farmers’ Welfare

B. Ministry of Micro, Small and Medium Enterprises

C. Ministry of Environment, Forest and Climate Change

D. None of the above

Answer: B

Explanation:

• The Ministry of Micro, Small and Medium Enterprises has flagged off ‘Apiary on Wheels’. ‘

• Apiary on Wheels’ is a unique concept designed by the  Khadi and Village Industries Commission

(KVIC) for the easy upkeep and migration of Bee Boxes having live Bee colonies.

• It is a holistic approach to address the challenges faced by the beekeepers.

• It is designed so as to reduce the labour and cost of maintaining and upkeeping Bee Boxes and live bee colonies across India.

www.YouTube.com/SleepyClasses www.SleepyClasses.com

48

Page 51: February · • The Regulating Act of 1773 created the office with the title of Governor-General of the Presidency of Fort William, or Governor-General of Bengal to be appointed

• Apiary on Wheels is a platform which can carry 20 Bee Boxes from one place to another without any difficulty.

• It is like an attachment which can be easily connected with a Tractor or a Trolley and may be pulled to any suitable destination.

10.Consider the statements regarding Madhav national park

1. It is located in Madhya Pradesh.

2. It is home to Particularly Vulnerable Tribal Groups (PVTG)s like Saharia.

Which of the above are correct?

A. 1 only

B. 2 only

C. Both 1 and 2

D. Neither 1 nor 2

Answer: C

Explanation:

• Madhav National Park  is situated in  Shivpuri District  of  Gwalior division  in northwest  Madhya

Pradesh, India.

• It was named after Madho Rao Scindia, the Maharaja of Gwalior belonging to the Scindia dynasty of

the Marathas.

• The Park is currently facing  displacement and rehabilitation  issues as it is home to  Particularly

Vulnerable Tribal Groups (PVTG)s like Saharia.

11.Consider the following statements regarding Intergovernmental Science-Policy Platform on Biodiversity and Ecosystem Services (IPBES)

1. It is an independent intergovernmental body established by States to strengthen the science-policy interface for biodiversity and ecosystem services.

2. It is a United Nations body.

3. United Nations Environment Programme (UNEP) provides secretariat services to IPBES.

Which of the above statements are correct?

A. 1 and 2 only

B. 1 and 3 only

C. 2 and 3 only

D. All of the above

Answer: B

www.YouTube.com/SleepyClasses www.SleepyClasses.com

49

Page 52: February · • The Regulating Act of 1773 created the office with the title of Governor-General of the Presidency of Fort William, or Governor-General of Bengal to be appointed

Explanation:

• The Intergovernmental Science-Policy Platform on Biodiversity and Ecosystem Services (IPBES) is an

independent intergovernmental body established by States to strengthen the science-policy interface for biodiversity and ecosystem services for the conservation and sustainable use of biodiversity, long-

term human well-being and sustainable development.

• It was established in Panama City, on 21 April 2012 by 94 Governments.

• It is not a United Nations body.

• However, at the request of the IPBES Plenary and with the authorization of the UNEP Governing Council in 2013, the United Nations Environment Programme (UNEP) provides secretariat services

to IPBES.

• It is based out of Bonn, Germany.

• IPBES currently has over 134 member States.

• A large number of NGOs, organizations, private sector conventions and civil society groupings also participate in the formal IPBES process as observers.

12.Consider the following statements w.r.t. Limiting Factors:

1. A limiting factor is a factor that restricts the size of a population from reaching its full potential.

2. Nitrogen is considered an important limiting factor in oceans.

3. Phosphates are the primary limiting factor in fresh water plant and algal growth.

Which of the above statements are correct?

A. 1 only

B. 1 and 2 only

C. 1 and 3 only

D. All of the above

Answer: D

Explanation:

• A  factor  present in an  environment  that controls a  process, particularly  growth, abundance or distribution of a population of organisms in an ecosystem.

• The term  limiting factor  is defined as a factor present in an environment that controls a process. In particular, it refers to the factor that limits  growth, abundance, and  distribution  of

a population of organisms in an ecosystem.

• Examples - Nitrogen is considered an important limiting factor in oceans.

• Phosphates are the primary limiting factor in fresh water plant and algal growth.

• Sunlight  in the rain forest, where growth is limited to all plants on the forest floor unless more light becomes available.

www.YouTube.com/SleepyClasses www.SleepyClasses.com

50

Page 53: February · • The Regulating Act of 1773 created the office with the title of Governor-General of the Presidency of Fort William, or Governor-General of Bengal to be appointed

13.Arrange the following components of Ecological Hierarchy in decreasing order

1. Biosphere

2. Ecosystem

3. Biome

4. Community

5. Population

Choose the correct option from the following:

A. 1 > 3 > 2 > 4 > 5

B. 1 > 3 > 2 > 5 > 4

C. 1 > 2 > 3 > 5 > 4

D. None of the above

Answer: A

Explanation:

• The first level of the ecological hierarchy is the individual organism. This level of the hierarchy

examines how one organism interacts with its environment.

• The second level involves populations. A population contains a group of individuals -- belonging to one species and living in a specific geographic area -- which interact with one another.

• The third level of the ecological hierarchy describes communities of life.

• The next level up is an ecosystem. A community is part of an ecosystem, but does not comprise an

entire ecosystem.

• Nonliving components in the environment are included in an ecosystem. The living organisms in an ecosystem interact with one another and with the nonliving factors in the environment.

• Examples of an ecosystem include a single lake, a confined forest, a prairie or a mountain summit.

• Then there are biomes and finally we have Biosphere which is at the widest level of analysis, the

biosphere represents the totality of all things on Earth, including their interactions.

14.Traits of an invasive species include

1. Fast growth

2. Rapid reproduction

3. High dispersal ability

4. Phenotypic plasticity

5. Tolerance of a wide range of environmental conditions.

Choose the correct option from the following

www.YouTube.com/SleepyClasses www.SleepyClasses.com

51

Page 54: February · • The Regulating Act of 1773 created the office with the title of Governor-General of the Presidency of Fort William, or Governor-General of Bengal to be appointed

A. 1, 2 and 3 only

B. 1, 2, 3 and 5 only

C. 4 and 5 only

D. All of the above

Answer: D

Explanation: An  invasive species  is a plant,  fungus, or animal  species  that is not native to a specific location (an  introduced species), and that has a tendency to spread to a degree believed to cause

damage to the environment, human economy or human health.

Common invasive species traits include the following -

• Fast growth

• Rapid reproduction

• High dispersal ability

• Phenotypic plasticity (the ability to alter growth form to suit current conditions)

• Tolerance of a wide range of environmental conditions (Ecological competence)

• Ability to live off of a wide range of food types (generalist)

• Association with humans

• Prior successful invasions

15.Consider the following statements regarding Ecotypes:

1. Species with wide geographical ranges which develop locally adapted populations.

2. They are also known as subspecies.

Which of the above statements is/are correct?

A. 1 only

B. 2 only

C. Both 1 and 2

D. None of the Above

Answer: A

Explanation:

• An ecotype, sometimes called ecospecies (not subspecies), describes a genetically distinct geographic variety, population or race within a species, which is genotypically adapted to specific environmental

conditions.

• Typically, though ecotypes exhibit  phenotypic  differences (such as in  morphology  or  physiology) stemming from environmental heterogeneity, they are capable of interbreeding with other

geographically adjacent ecotypes without loss of fertility or vigor. www.YouTube.com/SleepyClasses

www.SleepyClasses.com 52

Page 55: February · • The Regulating Act of 1773 created the office with the title of Governor-General of the Presidency of Fort William, or Governor-General of Bengal to be appointed

7. Science & Tech To watch the following questions on YouTube click on the links given below:

• Video 1

• Video 2

• Video 3

• Video 4

1. Government has recently abolished the anti-dumping duty on Purified Terephthalic Acid (PTA). PTA is

used in manufacturing of

A. Pesticides

B. Polyester production

C. Medicines

D. None of the above

Answer: B

Explanation:

• PTA is a raw material used in the production of polyester staple fibre and filaments

• It is critical input for textile fibres and yarns, and its easy availability at competitive prices is desirable to unlock immense potential in the textile sector, which is a significant employment generator

2. Which of following is/are correct in the context of pyrogen test?

1. It is carried out to check impurity or substance that can cause adverse side- effects

2. The test is performed on rats

Select the Correct Option

A. 1 only

B. 2 only

C. Both 1 and 2

D. Neither 1 nor 2

Answer: A

Explanation: The abnormal toxicity test is carried out to check potential hazardous biological

contamination in vaccine formulations.

www.YouTube.com/SleepyClasses www.SleepyClasses.com

53

Page 56: February · • The Regulating Act of 1773 created the office with the title of Governor-General of the Presidency of Fort William, or Governor-General of Bengal to be appointed

3. Which of the following statement(s) in the context of human body temperature are correct?

1. Researchers have found that the body temperatures have been declining over the last two

centuries

2. The decrease in temperature can be explained by reduction in metabolic rate

Select the Correct Option

A. 1 only

B. 2 only

C. Both 1 and 2

D. Neither 1 nor 2

Answer: C

Explanation: Different studies have found the human body temperature averaging out differently, including at 97.7°, 97.9° and 98.2°F.

Trends

• Body temperature is higher in younger people, in women, in larger bodies and at later times of the day

• Bodies of men born in the early to mid-1990s is on average 1.06°F cooler than those of men born in

the early 1800s

• Body temperature of women born in the early to mid-1990s is on average 0.58°F lower than that of

women born in the 1890s.

• Decrease in body temperature is the result of changes in the environment over the past 200 years,

which have in turn driven physiological changes

4. Which of the following are space-borne observatories of NASA?

1. Spitzer

2. Chandra X-Ray

3. Hubble

Select the Correct Option

A. 1 and 3 only www.YouTube.com/SleepyClasses

www.SleepyClasses.com 54

Page 57: February · • The Regulating Act of 1773 created the office with the title of Governor-General of the Presidency of Fort William, or Governor-General of Bengal to be appointed

B. 2 and 3 only

C. 1 and 2 only

D. All of the above

Answer: D

Explanation:

• NASA’s concept of Great Observatories is a series of four space-borne observatories designed to conduct astronomical studies over many different wavelengths (visible, gamma rays, X-rays, and

infrared)

• An important aspect of the Great Observatory program was to overlap the operations phases of the

missions to enable astronomers to make contemporaneous observations of an object at different

spectral wavelengths.

✓Hubble — UV, Visible and near-Infrared

✓Compton — Gamma Ray

✓Chandra — X-ray

✓Spitzer — Infrared

5. Which of the following statement(s) is/are correct in the context of Cancer in India?

1. There is a decreasing trend in the incidence rates of breast cancer across the country

2. The incidence rates of cervical cancer in India are increasing

Select the Correct Option:

A. 1 only

B. 2 only

C. Both 1 and 2

D. Neither 1 nor 2

Answer: D

Explanation:

• There is a clear increasing trend in the incidence rates of breast cancer across the country, with an annual percentage increase that ranges from 1.4% to 2.8% and is more pronounced in urban areas

than in rural areas.

• Incidence rates are also increasing for cancer types associated with overweight and obesity and lower

levels of physical activity, such as colorectal cancer, uterine cancer, ovarian cancer and prostate

cancer

www.YouTube.com/SleepyClasses www.SleepyClasses.com

55

Page 58: February · • The Regulating Act of 1773 created the office with the title of Governor-General of the Presidency of Fort William, or Governor-General of Bengal to be appointed

• There is a clear decreasing trend in the incidence rates of cervical cancer in most regions in India (annual percentage change, -2.0% to -3.5%), with age-standardized incidence rates as low as 6 per

1,00,000 in women in Kerala. India accounts for about one-fifth of the global burden of cervical cancer, despite decreasing incidence rates in several regions of the country.

6. Recently a new variety of rice, Muktoshri called IET 21845, has been developed. It is famous because

it is

A. drought-resistant

B. insect-resistant

C. arsenic-resistant

D. high-yielding variety

Answer: C

Explanation

• Researchers have developed and commercialised a rice variety that is resistant to arsenic. Several studies have shown that arsenic from groundwater and the soil can enter the food chain through

paddy

• The new rice variety, Muktoshri — also called IET 21845 —, was developed jointly by the Rice Research Station at Chinsurah coming under West Bengal’s Agriculture Department and the National

Botanical Research Institute, Lucknow

• According to the World Health Organization, long-term exposure to arsenic, mainly through drinking

water and food, can lead to poisoning. Skin lesions and skin cancer are the most characteristic effects

7. Which of the following states has banned CFL and filament bulbs from November 2020?

A. Kerala

B. Delhi

C. Haryana

D. Maharashtra

Answer: A

Explanation

• Kerala Finance Minister TM Thomas Isaac made a bold announcement that the state will impose a ban on the sale of compact fluorescent lamps (CFL) and incandescent (filament) bulbs starting November

this year as part of sustainable energy policy

• Streetlights and bulbs in government offices across the state will be converted to light-emitting diode (LED) bulbs

• The announcement is in line with the government project of Filament-free Kerala envisaged in 2018 as part of the state’s Urja Kerala mission

• Filament bulbs contain the mercury element which, when broken, is polluting in nature

www.YouTube.com/SleepyClasses www.SleepyClasses.com

56

Page 59: February · • The Regulating Act of 1773 created the office with the title of Governor-General of the Presidency of Fort William, or Governor-General of Bengal to be appointed

8. Which of the following correctly defines Dark Fibre?

A. Internet used for cyber-attacks

B. Unused optical fibre infra

C. Superfast internet speeds at night

D. None of the above

Answer: B

9. Adrain helmets were the first combat helmets issued by which of the following Army during World

War I?

A. France

B. Germany

C. England

D. Japan

Answer: A

Explanation:

• Adrain helmet, introduced by the French army in 1915 were the first combat helmet

• It is often described as the first modern steel helmet

• New research has found it performs better than modern military helmets in protecting the head from

shock waves created by overhead blasts

• In protection from ballistics and blunt impacts, modern military helmets have advanced. But in terms

of protecting the brain from shock waves from nearby blasts, the modern helmets are no better than

WWI helmets

10.Indradhanush is a joint military exercise between India and

A. Maldives

B. United Kingdom

C. United States

D. France

Answer: B

11.India will be hosting RAISE 2020 summit in the near future for social empowerment. It is related to which of the following?

A. Artificial Intelligence

B. Self-medication

C. Rape

www.YouTube.com/SleepyClasses www.SleepyClasses.com

57

Page 60: February · • The Regulating Act of 1773 created the office with the title of Governor-General of the Presidency of Fort William, or Governor-General of Bengal to be appointed

D. None of the above

Answer: A

Explanation:

• Ministry of Electronics and Information Technology is organizing the first Global AI Summit in India

(RAISE 2020), "RESPONSIBLE AI FOR SOCIAL EMPOWERMENT" with a focus on AI for Social

Transformation, Inclusion, and Empowerment

• It will bring together the innovators, investors and collaborators of the technology that will power the

future

• RAISE 2020 offers AI startups across the Globe a platform to connect, showcase, share their

expertise and exchange ideas with eminent Venture Capitalists, distinguished Global Leaders and

decision makers from the Government

12.H1N1 virus strain or swine flu causes the infection of

A. Respiratory System

B. Liver

C. Brain

D. Eyes

Answer: A

Explanation:

• First recognised in 1919, it is a human respiratory infection caused by an influenza strain that started

in pigs

• It may spread by

✓Droplet infection

✓skin-to-skin contact

✓Saliva

✓Touch

13.Melanin is the primary determinant of skin colour. It is found in

A. Hair

B. Eye

C. Ear

D. All of the above

Answer: D

www.YouTube.com/SleepyClasses www.SleepyClasses.com

58

Page 61: February · • The Regulating Act of 1773 created the office with the title of Governor-General of the Presidency of Fort William, or Governor-General of Bengal to be appointed

Explanation:

• It is found in hair, the pigmented tissue underlying the iris of the eye, and the stria vascularis of the

inner ear

• Melanin in skin is produced by melanocytes, found in the basal layer of the epidermis

• Some humans have very little or no melanin synthesis in their bodies, a condition called albinism

14.Raman effect is related to

A. Scattering of light

B. Interference of light

C. Total Internal Reflection

D. None of the above

Answer: A

Explanation:

• India celebrates National Science Day on 28 February every year in honour of physicist C.V. Raman’s discovery of the Raman Effect, which gave Asia its first Nobel in the Sciences in 1930

• Raman Effect is the process of scattering of light particles by molecules of a medium. The scattering

occurs due to a change in the wavelength of light as it enters the medium. When a beam of light travels through a dust-free, transparent chemical, a small fraction of the light emerges in directions

other than where it should

www.YouTube.com/SleepyClasses www.SleepyClasses.com

59